Ch 27 - Coronary Vascular Disease

Réussis tes devoirs et examens dès maintenant avec Quizwiz!

The nurse is educating a patient diagnosed with angina pectoris about the difference between the pain of angina and a myocardial infarction (MI). How should the nurse describe the pain experienced during an MI? (Select all that apply.) It is viselike and radiates to the shoulders and arms. It is substernal in location. It is sudden in onset and prolonged in duration. It is relieved by rest and inactivity. It subsides after taking nitroglycerin.

Ans: It is substernal in location. It is sudden in onset and prolonged in duration. It is viselike and radiates to the shoulders and arms. Feedback: Chest pain that occurs suddenly, continues despite rest and medication, is substernal, and is sometimes viselike and radiating to the shoulders and arms is associated with an MI. Angina pectoris pain is generally relieved by rest and nitroglycerin.

The nurse is explaining the cause of angina pain to a client. What will the nurse say caused the pain? a lack of oxygen in the heart muscle that causes the death of cells complete closure of an artery a destroyed part of the heart muscle incomplete blockage of a major coronary artery

Ans: a lack of oxygen in the heart muscle that causes the death of cells Feedback: Impeded blood flow, due to blockage in a coronary artery, deprives the cardiac muscle cells of oxygen, thus leading to a condition known as ischemia. Artery blockage or closure leads to myocardial death. The destroyed part of the heart is a myocardial infarction.

A client has recently undergone a coronary artery bypass graft (CABG). The nurse should be alert to which respiratory complication? Elevated blood glucose level Urinary tract infection (UTI) Hyperkalemia Atelectasis

Ans: Atelectasis Feedback: Respiratory complications that may occur include atelectasis. An incentive spirometer and the use of deep breathing exercises are necessary to prevent atelectasis and pneumonia. Elevated blood sugar levels, hyperkalemia, UTI, and are complications that can occur but are unrelated to the respiratory system.

The nurse is to administer morphine sulfate to a client with chest pain. What initial nursing action is required prior to administration? Measure the blood pressure for hypertension. Count the respiratory rate for bradypnea. Check the radial pulse for dysrhythmias. Measure urinary output for dehydration.

Ans: Count the respiratory rate for bradypnea. Feedback: The nurse should always check the respiratory rate prior to administering morphine sulfate. The drug should be withheld, and the health care provider notified, if the respiratory rate is below 16 breaths/minute.

The nurse is caring for a client with coronary artery disease (CAD). What is an appropriate nursing action when evaluating a client with CAD? a) Assess for any kind of drug abuse. b) Assess the characteristics of chest pain. c) Assess the client's mental and emotional status. d) Assess the skin of the client.

Ans: Assess the characteristics of chest pain. Feedback: The nurse should assess the characteristics of chest pain for a client with CAD. Assessing the client's mental and emotional status, skin, or for drug abuse will not assist the nurse in evaluating the client for CAD. The assessment should be aimed at evaluating for adequate blood flow to the heart.

The ED nurse is caring for a patient with a suspected MI. What drug should the nurse anticipate administering to this patient? A) Oxycodone B) Warfarin C) Morphine D) Acetaminophen

Ans: Morphine Feedback: The patient with suspected MI is given aspirin, nitroglycerin, morphine, an IV beta- blocker, and other medications, as indicated, while the diagnosis is being confirmed. Tylenol, warfarin, and oxycodone are not typically used.

A patient with angina has been prescribed nitroglycerin. Before administering the drug, the nurse should inform the patient about what potential adverse effects? A) Nervousness or paresthesia B) Throbbing headache or dizziness C) Drowsiness or blurred vision D) Tinnitus or diplopia

Ans: Throbbing headache or dizziness Feedback: Headache and dizziness commonly occur when nitroglycerin is taken at the beginning of therapy. Nervousness, paresthesia, drowsiness, blurred vision, tinnitus, and diplopia do not typically occur as a result of nitroglycerin therapy.

The nurse is reviewing the results of a total cholesterol level for a client who has been taking simvastatin. What results display the effectiveness of the medication? 160-190 mg/dL 210-240 mg/dL 250-275 mg/dL 280-300 mg/dL

Ans: 160-190 mg/dL Feedback: Simvastatin is a statin frequently given as initial therapy for significantly elevated cholesterol and low-density lipoprotein levels. Normal total cholesterol is less than 200 mg/dL.

Which is the analgesic of choice for acute myocardial infarction (MI)? Morphine Aspirin Meperidine Ibuprofen

Ans: Morphine Feedback: The analgesic of choice for acute MI is morphine administered in IV boluses to reduce pain and anxiety. Aspirin is an antiplatelet medication. Meperidine and Ibuprofen are not the analgesics of choice.

The nurse is assessing a client with severe angina pectoris and electrocardiogram changes in the emergency room. What is the most important cardiac marker for the client? creatine kinase lactate dehydrogenase myoglobin troponin

Ans: troponin Feedback: This client exhibits signs of myocardial infarction (MI), and the most accurate serum determinant of an MI is troponin level. Creatine kinase, lactate dehydrogenase, and myoglobin tests can show evidence of muscle injury, but the studies are less specific indicators of myocardial damage than troponin.

An OR nurse is preparing to assist with a coronary artery bypass graft (CABG). The OR nurse knows that the vessel most commonly used as source for a CABG is what? A) Brachial artery B) Brachial vein C) Femoral artery D) Greater saphenous vein

Ans: Greater saphenous vein Feedback: The greater saphenous vein is the most commonly used graft site for CABG. The right and left internal mammary arteries, radial arteries, and gastroepiploic artery are other graft sites used, though not as frequently. The femoral artery, brachial artery, and brachial vein are never harvested.

The nurse is caring for a client experiencing an acute MI (STEMI). The nurse anticipates the physician will prescribe alteplase. Before administering this medication, which question is most important for the nurse to ask the client? "Do your parents have heart disease?" "What time did your chest pain start today?" "How many sublingual nitroglycerin tablets did you take?" "What is your pain level on a scale of 1 to 10?"

Ans: "What time did your chest pain start today?" Feedback: The client may be a candidate for thrombolytic (fibrolytic) therapy. These medications are administered if the client's chest pain lasts longer than 20 minutes and is unrelieved by nitroglycerin, if ST-segment elevation is found in at least two leads that face the same area of the heart, and if it has been less than 6 hours since the onset of pain. The most appropriate question for the nurse to ask is in relations to when the chest pain began. The other questions would not aid in determining whether the client is a candidate for thrombolytic therapy.

A client has just arrived in the ER with a possible myocardial infarction (MI). The electrocardiogram (ECG) should be obtained within which time frame of arrival to the ER? 15 minutes 5 minutes 10 minutes 20 minutes

Ans: 10 minutes Feedback: The ECG provides information that assists in diagnosing acute MI. It should be obtained within 10 minutes from the time a client reports pain or arrives in the emergency department. By monitoring serial ECG changes over time, the location, evolution, and resolution of an MI can be identified and monitored.

Following a percutaneous transluminal coronary angioplasty, a client is monitored in the postprocedure unit. The client's heparin infusion was stopped 2 hours earlier. There is no evidence of bleeding or hematoma at the insertion site, and the pressure device is removed. With regards to partial thromboplastin time (PTT), when should the nurse plan to remove the femoral sheath? 100 seconds or less. 75 seconds or less. 125 seconds or less. 50 seconds or less.

Ans: 50 seconds or less. Feedback: Heparin causes an elevation of the PTT and, thereby, increases the risk for bleeding. With a large cannulation such as a sheath used for angioplasty, the PTT should be 50 seconds or less before the sheath is removed. Removing the sheath before the PTT drops below 50 seconds can cause bleeding at the insertion site. The other PTT results are incorrect for determining when to remove the sheath.

To be effective, percutaneous transluminal coronary angioplasty (PTCA) must be performed within what time frame, beginning with arrival at the emergency department after diagnosis of myocardial infarction (MI)? 30 minutes 60 minutes 9 days 6 to 12 months

Ans: 60 minutes Feedback: The 60-minute interval is known as "door-to-balloon time" in which a PTCA can be performed on a client with a diagnosed MI. The 30-minute interval is known as "door-to-needle time" for the administration of thrombolytics after MI. The time frame of 9 days refers to the time until the onset of vasculitis after administration of streptokinase for thrombolysis in a client with an acute MI. The 6- to 12-month time frame refers to the time period during which streptokinase will not be used again in the same client for acute MI.

The charge nurse was discussing with the nursing student that studies have been published that suggest inflammation increases the risk of heart disease. Which modifiable factor would the nursing student target in teaching clients about prevention of inflammation that can lead to atherosclerosis? Drink at least 2 liters of water a day Avoid use of caffeine Addressing obesity Encourage use of a multivitamin

Ans: Addressing obesity Feedback: Published information by Balistreri et al. (2010) indicated a relationship between body fat and the production of inflammatory and thrombotic (clot-facilitating) proteins. This information suggests decreasing obesity and body fat stores may help to reduce the risk. Avoiding the use of caffeine, encouraging the use of a multivitamin, and drinking at least 2 liters of water a day are not actions that will address the prevention of inflammation that can lead to atherosclerosis.

A client in the emergency department reports squeezing substernal pain that radiates to the left shoulder and jaw. The client also complains of nausea, diaphoresis, and shortness of breath. What is the nurse's priority action? Complete the client's registration information, perform an electrocardiogram, gain I.V. access, and take vital signs. Alert the cardiac catheterization team, administer oxygen, attach a cardiac monitor, and notify the health care provider. Gain I.V. access, give sublingual nitroglycerin, and alert the cardiac catheterization team. Administer oxygen, attach a cardiac monitor, take vital signs, and administer sublingual nitroglycerin.

Ans: Administer oxygen, attach a cardiac monitor, take vital signs, and administer sublingual nitroglycerin. Feedback: Cardiac chest pain is caused by myocardial ischemia. Therefore the nurse should administer supplemental oxygen to increase the myocardial oxygen supply, attach a cardiac monitor to help detect life-threatening arrhythmias, and take vital signs to ensure that the client isn't hypotensive before giving sublingual nitroglycerin for chest pain. Registration information may be delayed until the client is stabilized. Alerting the cardiac catheterization team or the health care provider before completing the initial assessment is premature.

An older adult is postoperative day one, following a coronary artery bypass graft (CABG). The client's family members express concern to the nurse that the client is uncharacteristically confused. After reporting this change in status to the health care provider, what additional action should the nurse take? Educate the family about how confusion is expected in older adults postoperatively. Assess for factors that may be causing the client's delirium. Document the early signs of dementia and ensure the client's safety. Reorient the client to place and time.

Ans: Assess for factors that may be causing the client's delirium. Feedback: Uncharacteristic changes in cognition following cardiac surgery are suggestive of delirium. Dementia has a gradual onset with organic brain changes and is not an acute response to surgery. Assessment is a higher priority than reorientation, which may or may not be beneficial. Even though delirium is not rare, it is not considered to be an expected part of recovery.

The nurse is providing education about angina pectoris to a hospitalized client who is about to be discharged. What instruction does the nurse include about managing this condition? Select all that apply. Carry nitroglycerin at all times. Avoid all physical activity. Balance rest with activity. Follow a diet high in saturated fats. Stop smoking.

Ans: Balance rest with activity. Stop smoking. Carry nitroglycerin at all times. Feedback: Managing angina pectoris at home includes balancing rest with activity, participating in a regular daily program of activities that do not induce angina pain, stopping smoking, carrying nitroglycerin at all times, and following a diet low in saturated fat.

Which of the following is inconsistent as a condition related to metabolic syndrome? Abdominal obesity Hypotension Insulin resistance Dyslipidemia

Ans: Hypotension Feedback: A diagnosis of metabolic syndrome includes three of the following conditions: insulin resistance, abdominal obesity, dyslipidemia, hypertension, proinflammatory state, and prothrombotic state.

Which technique is used to surgically revascularize the myocardium? Peripheral bypass Minimally invasive direct coronary bypass Gastric bypass Balloon bypass

Ans: Minimally invasive direct coronary bypass Feedback: Several techniques are used to surgically revascularize the myocardium; one of them is minimally invasive direct coronary bypass. Balloon bypass is not used to revascularize the myocardium. If the client is experiencing acute pain in the leg, peripheral bypass is performed.

A triage team is assessing a client to determine if reported chest pain is a manifestation of angina pectoris or an MI. The nurse knows that a primary distinction of angina pain is? Described as crushing and substernal Relieved by rest and nitroglycerin Associated with nausea and vomiting Accompanied by diaphoresis and dyspnea

Ans: Relieved by rest and nitroglycerin Feedback: One characteristic that can differentiate the pain of angina from a myocardial infarction is pain that is relieved by rest and nitroglycerine. There may be some exceptions (unstable angina), but the distinction is helpful especially when combined with other assessment data.

A client's lipid profile reveals an LDL level of 122 mg/dL. This is considered a: low LDL level. fasting LDL level. high LDL level. normal LDL level.

Ans: high LDL level. Feedback: LDL levels above 100 mg/dL are considered high. The goal is to decrease the LDL level below 100 mg/dL.

BELOW ARE CH 27 PREPU

Ans: Feedback:

A client is admitted for treatment of Prinzmetal's angina. When developing this client's care plan, the nurse should keep in mind that this type of angina is a result of what trigger? An unpredictable amount of activity. Coronary artery spasm. Activities that increase myocardial oxygen demand. The same type of activity that caused previous angina episodes.

Ans: Coronary artery spasm. Feedback: Prinzmetal's angina results from coronary artery spasm. Activities that increase myocardial oxygen demand may trigger angina of effort. An unpredictable amount of activity may precipitate unstable angina. Worsening angina is brought on by the same type or level of activity that caused previous angina episodes; anginal pain becomes increasingly severe.

In the treatment of coronary artery disease (CAD), medications are often ordered to control blood pressure in the client. Which of the following is a primary purpose of using beta-adrenergic blockers in the nursing management of CAD? To decrease homocysteine levels To prevent angiotensin II conversion To dilate coronary arteries To decrease workload of the heart

Ans: To decrease workload of the heart Feedback: Beta-adrenergic blockers are used in the treatment of CAD to decrease the myocardial oxygen by reducing heart rate and workload of the heart. Nitrates are used for vasodilation. Anti-lipid drugs (such as statins and B vitamins) are used to decrease homocysteine levels. ACE inhibitors inhibit the conversion of angiotensin.

The nurse is administering a calcium channel blocker to a patient who has symptomatic sinus tachycardia at a rate of 132 bpm. What is the anticipated action of the drug for this patient? Decreases the sinoatrial node automaticity Increases the atrioventricular node conduction Increases the heart rate Creates a positive inotropic effect

Ans: Decreases the sinoatrial node automaticity Feedback: Calcium channel blockers have a variety of effects on the ischemic myocardium. These agents decrease sinoatrial node automaticity and atrioventricular node conduction, resulting in a slower heart rate and a decrease in the strength of myocardial contraction (negative inotropic effect).

A client comes to the emergency department reporting chest pain. An electrocardiogram (ECG) reveals myocardial ischemia and an anterior-wall myocardial infarction (MI). Which ECG characteristic does the nurse expect to see? Prolonged PR interval Absent Q wave Elevated ST segment Widened QRS complex

Ans: Elevated ST segment Feedback: Ischemic myocardial tissue changes cause elevation of the ST segment, an inverted T wave, and a pathological Q wave. A prolonged PR interval occurs with first-degree heart block, the least dangerous atrioventricular heart block; this disorder may arise in healthy people but sometimes results from drug toxicity, electrolyte or metabolic disturbances, rheumatic fever, or chronic degenerative disease of the conduction system. An absent Q wave is normal; an MI may cause a significant Q wave. A widened QRS complex indicates a conduction delay in the His-Purkinje system.

A nurse is teaching about risk factors that increase the probability of heart disease to a community group. Which risk factors will the nurse include in the discussion? Select all that apply. Age greater than 45 years for men Body mass index (BMI) of 23 African-American descent Family history of coronary heart disease Elevated C-reactive protein

Ans: Family history of coronary heart disease Age greater than 45 years for men African-American descent Elevated C-reactive protein Feedback: Risk factors for coronary heart disease (CHD) include family history of CHD, age older than 45 years for men and 65 years for women, African-American race, BMI of 25 or greater, and elevated C-reactive protein.

The nurse is discussing risk factors for developing CAD with a patient in the clinic. Which results would indicate that the patient is not at significant risk for the development of CAD? Low density lipoprotein (LDL), 160 mg/dL Cholesterol, 280 mg/dL A ratio of LDL to HDL, 4.5 to 1.0 High-density lipoprotein (HDL), 80 mg/dL

Ans: High-density lipoprotein (HDL), 80 mg/dL Feedback: A fasting lipid profile should demonstrate the following values (Alberti et al., 2009): LDL cholesterol less than 100 mg/dL (less than 70 mg/dL for very high-risk patients); total cholesterol less than 200 mg/dL; HDL cholesterol greater than 40 mg/dL for males and greater than 50 mg/dL for females; and triglycerides less than 150 mg/dL.

Understanding atherosclerosis, the nurse identifies which of the following to be both a risk factor for the development of the disorder and an outcome? Hyperlipidemia Hypertension Glucose intolerance Obesity

Ans: Hypertension Feedback: Increases in diastolic and systolic blood pressure are associated with an increased incidence of atherosclerosis, often an inherited factor. Elevation of blood pressure results when the vessels cannot relax and impairs the ability of the artery to dilate. Hyperlipidemia, diabetes, and obesity are all risk factors for atherosclerosis but do not result from the disorder.

An electrocardiogram (ECG) taken during a routine checkup reveals that a client has had a silent myocardial infarction. Changes in which leads of a 12-lead ECG indicate damage to the left ventricular septal region? Leads I, aVL, V5, and V6 Leads V1 and V2 Leads V3 and V4 Leads II, III, and aVF

Ans: Leads V3 and V4 Feedback: Leads V3 and V4 record electrical events in the septal region of the left ventricle. Leads I, aVL, V5, and V6 record electrical events on the lateral surface of the left ventricle. Leads II, III, and aVF record electrical events on the inferior surface of the left ventricle. Leads V1 and V2 record electrical events on the anterior surface of the right ventricle and the anterior surface of the left ventricle.

The nurse is caring for a client after cardiac surgery. What laboratory result will lead the nurse to suspect possible renal failure? an hourly urine output of 50 to 70 mL a urine specific gravity reading of 1.021 a serum BUN of 70 mg/dL a serum creatinine of 1.0 mg/dL

Ans: a serum BUN of 70 mg/dL Feedback: These four laboratory results should always be assessed after cardiac surgery. Serum osmolality (N = >800 mOsm/kg) should also be included. A BUN reading of greater than 21 mg/dL is abnormal; a reading of greater than 60 mg/dL is indicative of renal failure. Urine output needs to be greater than 30 mL/hr. Normal urine specific gravity is 1.005-1.030. Normal serum creatinine values are between 0.5-1.2 mg/dL.

The nurse is caring for a ventilated client after coronary artery bypass graft surgery. What are the criterions for extubation for the client? Select all that apply. acceptable arterial blood gas values inability to speak adequate cough and gag reflexes labile vital signs breathing without assistance of the ventilator

Ans: adequate cough and gag reflexes acceptable arterial blood gas values breathing without assistance of the ventilator Feedback: Before being extubated, the client should have cough and gag reflexes and stable vital signs; be able to lift the head off the bed or give firm hand grasps; have adequate vital capacity, negative inspiratory force, and minute volume appropriate for body size; and have acceptable arterial blood gas levels while breathing without the assistance of the ventilator. Inability to talk is expected when intubated with an endotracheal tube.

A client who has been diagnosed with arteriosclerosis is confused by what this means. The nurse explains that arteriosclerosis is: a condition in which the lumen of arteries fill with scar tissue. an expected part of the aging process. high level of blood fat. a vascular occlusive disease.

Ans: an expected part of the aging process. Feedback: Arteriosclerosis is loss of elasticity or hardening of the arteries that accompanies the aging process. While arteriosclerosis is a contributing factor to vascular occlusive disease, it is a term that refers to a loss of elasticity or hardening of the arteries that accompanies the aging process. Arteriosclerosis does not involve scar tissue formation. Hyperlipidemia, or high levels of blood fat, triggers atherosclerotic changes.

A client with a myocardial infarction (MI) develops pulmonary crackles and dyspnea. A chest X-ray shows evidence of pulmonary edema. What type of MI did this client have? inferior. anterior. posterior. lateral.

Ans: anterior. Feedback: An anterior MI causes left ventricular dysfunction and can lead to manifestations of heart failure, which include pulmonary crackles and dyspnea. Posterior, lateral, and inferior MI aren't usually associated with heart failure.

The nurse is caring for a client diagnosed with coronary artery disease (CAD). What condition most commonly results in CAD? myocardial infarction diabetes mellitus atherosclerosis renal failure

Ans: atherosclerosis Feedback: Atherosclerosis (plaque formation) is the leading cause of CAD. Diabetes mellitus is a risk factor for CAD, but it isn't the most common cause. Myocardial infarction is a common result of CAD. Renal failure doesn't cause CAD, but the two conditions are related.

A client with CAD thinks diltiazem (Cardizem) has been causing nausea. Diltiazem (Cardizem) is categorized as which type of drug? calcium-channel blocker beta-adrenergic blocker nitrate diuretic

Ans: calcium-channel blocker Feedback: Calcium-channel blocking agents may be used to treat CAD as well, although research has shown that they may be less beneficial than beta-adrenergic blocking agents. Diltiazem (Cardizem) is an example of a calcium-channel blocker.

A client who has been diagnosed with Prinzmetal's angina will present with which symptom? chest pain of increased frequency, severity, and duration prolonged chest pain that accompanies exercise radiating chest pain that lasts 15 minutes or less chest pain that occurs at rest and usually in the middle of the night

Ans: chest pain that occurs at rest and usually in the middle of the night Feedback: A client with Prinzmetal's angina will complain of chest pain that occurs at rest, usually between 12 and 8:00 AM, is sporadic over 3-6 months, and diminishes over time. Clients with stable angina generally experience chest pain that lasts 15 minutes or less and may radiate. Clients with Cardiac Syndrome X experience prolonged chest pain that accompanies exercise and is not always relieved by medication. Clients with unstable angina experience chest pain of increased frequency, severity, and duration that is poorly relieved by rest or oral nitrates.

The nurse is caring for a client with coronary artery disease. What is the nurse's priority goal for the client? decrease anxiety administer sublingual nitroglycerin educate the client about his symptoms enhance myocardial oxygenation

Ans: enhance myocardial oxygenation Feedback: Enhancing myocardial oxygenation is always the first priority when a client exhibits signs or symptoms of cardiac compromise. Without adequate oxygen, the myocardium suffers damage. A nurse administers sublingual nitroglycerin to treat acute angina pectoris, but its administration isn't the first priority. Although educating the client and decreasing anxiety are important in care, neither is a priority when a client is compromised.

The laboratory values for a client diagnosed with coronary artery disease (CAD) have just come back from the lab. The client's low-density lipoprotein (LDL) level is 112 mg/dL. This nurses recognizes that this value is high. extremely high. normal. low.

Ans: high. Feedback: If the LDL level ranges from 100 mg/dL to 130 mg/dL, it is considered to be high. The goal is to decrease the LDL level below 100 mg/dL.

The nurse is reviewing the laboratory results for a client diagnosed with coronary artery disease (CAD). The client's low-density lipoprotein (LDL) level is 115 mg/dL. The nurse interprets this value as low. high. within normal limits. critically high.

Ans: high. Feedback: The normal LDL range is 100 to 130 mg/dL. A level of 115 mg/dL is considered to be high. The goal of treatment is to decrease the LDL level below 100 mg/dL (less than 70 mg/dL for very high-risk clients).

A client with a family history of coronary artery disease reports experiencing chest pain and palpitations during and after morning jogs. What would reduce the client's cardiac risk? smoking cessation a protein-rich diet exercise avoidance antioxidant supplements

Ans: smoking cessation Feedback: The first line of defense for clients with CAD is lifestyle changes including smoking cessation, weight loss, stress management, and exercise. Clients with CAD should eat a balanced diet. Clients with CAD should exercise, as tolerated, to maintain a healthy weight. Antioxidant supplements, such as those containing vitamin E, beta carotene, and selenium, are not recommended because clinical trials have failed to confirm beneficial effects from their use.

The nurse is administering oral metoprolol. Where are the receptor sites mainly located? Heart Uterus Bronchi Blood vessels

Ans: Heart Feedback: Metoprolol works at beta1 receptor sites. Most beta1 receptor sites are located in the heart. Beta2 receptors are located in the uterus, blood vessels, and bronchi

When the nurse notes that, after cardiac surgery, the client demonstrates low urine output (less than 25 mL/h) with high specific gravity (greater than 1.025), the nurse suspects which condition? Anuria Inadequate fluid volume Normal glomerular filtration Overhydration

Ans: Inadequate fluid volume Explanation: Urine output less than 0.5 mL/kg/h may indicate a decrease in cardiac output. A high specific gravity indicates increased concentration of solutes in the urine, which occurs with inadequate fluid volume. Indices of normal glomerular filtration are output of 0.5 mL/kg/h or more and specific gravity between 1.010 and 1.025. Overhydration is manifested by high urine output with low specific gravity. The anuric client does not produce urine.

A client with angina pectoris must learn how to reduce risk factors that exacerbate this condition. When developing the client's care plan, which expected outcome should a nurse include? "Client will verbalize an understanding of the need to call the physician if acute pain lasts more than 2 hours." "Client will verbalize the intention to stop smoking." "Client will verbalize an understanding of the need to restrict dietary fat, fiber, and cholesterol." "Client will verbalize the intention to avoid exercise."

Ans: "Client will verbalize the intention to stop smoking." Feedback: A client with angina pectoris should stop smoking at once because smoking increases the blood carboxyhemoglobin level; this increase, in turn, reduces the heart's oxygen supply and may induce angina. The client must seek immediate medical attention if chest pain doesn't subside after three nitroglycerin doses taken 5 minutes apart; serious myocardial damage or even sudden death may occur if chest pain persists for 2 hours. To improve coronary circulation and promote weight management, the client should get regular daily exercise. The client should eat plenty of fiber, which may decrease serum cholesterol and triglyceride levels and minimize hypertension, in turn reducing the risk for atherosclerosis (which plays a role in angina).

A client, who has undergone a percutaneous transluminal coronary angioplasty (PTCA), has received discharge instructions. Which statement by the client would indicate the need for further teaching by the nurse? "I should avoid prolonged sitting." "I should expect a low-grade fever and swelling at the site for the next week." "I should expect bruising at the catheter site for up to 3 weeks." "I should avoid taking a tub bath until my catheter site heals."

Ans: "I should expect a low-grade fever and swelling at the site for the next week." Feedback: Fever and swelling at the site are signs of infection and should be reported to the physician. Showers should be taken until the insertion site is healed. Prolonged sitting can result in thrombosis formation. Bruising at the insertion site is common and may take from 1 to 3 weeks to resolve.

When providing discharge instructions for a client who has been prescribed sublingual nitroglycerin for angina, the nurse should plan to include which instructions? "Place the nitroglycerin tablet between cheek and gum." "See if rest relieves the chest pain before using the nitroglycerin." "Only take one nitroglycerin tablet for each episode of angina." "Call 911 if you develop a headache following nitroglycerin use."

Ans: "See if rest relieves the chest pain before using the nitroglycerin." Feedback: Decreased activity may relieve chest pain; sitting will prevent injury should the nitroglycerin lower BP and cause fainting. The client should expect to feel dizzy or flushed or to develop a headache following sublingual nitroglycerin use. The client should place one nitroglycerin tablet under the tongue if 2-3 minutes of rest fails to relieve pain. Clients may take up to three nitroglycerin tablets within 5 minutes of each other to relieve angina. However, they should call 911 if the three tablets fail to resolve the chest pain.

Heparin therapy is usually considered therapeutic when the client's activated partial thromboplastin time (aPTT) is how many times normal? .25 to .75 .75 to 1.5 2.0 to 2.5 2.5 to 3.0

Ans: 2.0 to 2.5 Feedback: The amount of heparin administered is based on aPTT results, which should be obtained during the follow-up to any alteration of dosage. The client's aPTT value would have to be greater than .25 to .75 or .75 to 1.5 times normal to be considered therapeutic. An aPTT value that is 2.5 to 3 times normal would be too high to be considered therapeutic.

A client asks the nurse how long to wait after taking nitroglycerin before experiencing pain relief. What is the best answer by the nurse? 15 minutes 30 minutes 60 minutes 3 minutes

Ans: 3 minutes Feedback: Nitroglycerin may be given by several routes: sublingual tablet or spray, oral capsule, topical agent, and intravenous (IV) administration. Sublingual nitroglycerin is generally placed under the tongue or in the cheek (buccal pouch) and ideally alleviates the pain of ischemia within 3 minutes.

The nurse is teaching a client with suspected acute myocardial infarction about serial isoenzyme testing. When is it best to have isoenzyme creatinine kinase of myocardial muscle (CK-MB) tested? 30 minutes to 1 hour after pain 2 to 3 hours after admission 4 to 6 hours after pain 12 to 18 hours after admission

Ans: 4 to 6 hours after pain Feedback: Serum CK-MB levels can be detected 4 to 6 hours after the onset of chest pain. These levels peak within 12 to 18 hours and return to normal within 3 to 4 days.

A nurse is reevaluating a client receiving IV fibrinolytic therapy. Which finding requires immediate intervention by the nurse? Altered level of consciousness Minimal oozing of blood from the IV site Presence of reperfusion dysrhythmias Chest pain 2 of 10 (on a 1-to-10 pain scale)

Ans: Altered level of consciousness Feedback: A client receiving fibrinolytic therapy is at risk for complications associated with bleeding. Altered level of consciousness may indicate hypoxia and intracranial bleeding, and the infusion should be discontinued immediately. Minimal bleeding requires manual pressure. Reperfusion dysrhythmias are an expected finding. A chest pain score of 2 is low and indicates the client's chest pain is subsiding, an expected outcome of this therapy.

Which method to induce hemostasis after sheath removal after percutaneous transluminal coronary angioplasty (PTCA) is most effective? Application of a sandbag to the area Application of a vascular closure device such as Angio-Seal or VasoSeal Application of a pneumatic compression device (e.g., FemoStop) Direct manual pressure

Ans: Application of a vascular closure device such as Angio-Seal or VasoSeal Feedback: Application of a vascular closure device has been demonstrated to be very effective. Direct manual pressure to the sheath introduction site and application of a pneumatic compression device after PTCA have been demonstrated to be effective; the former was the first method used to induce hemostasis after PTCA. Several nursing interventions frequently used as part of the standard of care, such as applying a sandbag to the sheath insertion site, have not been shown to be effective in reducing the incidence of bleeding.

The nurse is presenting a workshop at the senior citizens center about how the changes of aging predispose clients to vascular occlusive disorders. What would the nurse name as the most common cause of peripheral arterial problems in the older adult? Atherosclerosis Coronary thrombosis Raynaud's disease Arteriosclerosis

Ans: Atherosclerosis Feedback: Atherosclerosis is the most common cause of peripheral arterial problems in the older adult. The disease correlates with the aging process. The other choices may occur at any age.

The nurse administers propranolol hydrochloride to a patient with a heart rate of 64 beats per minute (bpm). One hour later, the nurse observes the heart rate on the monitor to be 36 bpm. What medication should the nurse prepare to administer that is an antidote for the propranolol? Sodium nitroprusside Digoxin Protamine sulfate Atropine

Ans: Atropine Feedback: Sheath removal and the application of pressure on the vessel insertion site may cause the heart rate to slow and the blood pressure to decrease (vasovagal response). A dose of IV atropine is usually given to treat this response.

A patient has had cardiac surgery and is being monitored in the intensive care unit (ICU). What complication should the nurse monitor for that is associated with an alteration in preload? Hypothermia Elevated central venous pressure Cardiac tamponade Hypertension

Ans: Cardiac tamponade Feedback: Preload alterations occur when too little blood volume returns to the heart as a result of persistent bleeding and hypovolemia. Excessive postoperative bleeding can lead to decreased intravascular volume, hypotension, and low cardiac output. Bleeding problems are common after cardiac surgery because of the effects of cardiopulmonary bypass, trauma from the surgery, and anticoagulation. Preload can also decrease if there is a collection of fluid and blood in the pericardium (cardiac tamponade), which impedes cardiac filling. Cardiac output is also altered if too much volume returns to the heart, causing fluid overload.

A nurse is caring for a client in the cardiovascular intensive care unit following a coronary artery bypass graft. Which clinical finding requires immediate intervention by the nurse? Central venous pressure reading of 1 Blood pressure 110/68 mm Hg Heart rate 66 bpm Pain score 5/10

Ans: Central venous pressure reading of 1 Feedback: The central venous pressure (CVP) reading of 1 is low (2-6 mm Hg) and indicates reduced right ventricular preload, commonly caused by hypovolemia. Hypovolemia is the most common cause of decreased cardiac output after cardiac surgery. Replacement fluids such as colloids, packed red blood cells, or crystalloid solutions may be prescribed. The other findings require follow-up by the nurse; however, addressing the CVP reading is the nurse's priority.

The nurse is assisting with a bronchoscopy at the bedside in a critical care unit. The client experiences a vasovagal response. What should the nurse do next? Prepare to administer intravenous fluids. Check blood pressure. Assess pupils for reactiveness. Suction the airway.

Ans: Check blood pressure. Feedback: During a bronchoscopy, a vasovagal response may be caused by stimulating the pharynx, and it in turn may cause stimulation of the vagus nerve. The client may, therefore, experience a sudden drop in heart rate, leading to syncope. The nurse will need to assess blood pressure to assure circulation. Stimulation of the vagus nerve doesn't lead to pupillary dilation or bronchodilation. Stimulation of the vagus nerve increases gastric secretions.

A client had a percutaneous transluminal coronary angioplasty (PTCA). What medication will the nurse administer to prevent thrombus formation in the stent? Isosorbide mononitrate Diltiazem Clopidogrel Metoprolol

Ans: Clopidogrel Feedback: Because of the risk of thrombus formation following a coronary stent placement, the patient receives antiplatelet medications, such as clopidogrel or aspirin. Isosorbide mononitrate is a nitrate used for vasodilation. Metoprolol is a beta blocker used for relaxing blood vessels and slowing heart rate. Diltiazem is a calcium channel blocker used to relax heart muscles and blood vessels.

The nurse is caring for a client who was admitted to the telemetry unit with a diagnosis of "rule/out acute MI." The client's chest pain began 3 hours earlier. Which laboratory test would be most helpful in confirming the diagnosis of a current MI? CK-MM Creatinine kinase-myoglobin (CK-MB) level Troponin C level Myoglobin level

Ans: Creatinine kinase-myoglobin (CK-MB) level Feedback: Elevated CK-MB assessment by mass assay is an indicator of acute MI; the levels begin to increase within a few hours and peak within 24 hours of an MI. If the area is reperfused (as a result of thrombotic therapy or PCI), it peaks earlier. CK-MM (skeletal muscle) is not an indicator of cardiac muscle damage. Three isomers of troponin exist: C, I, and T. Troponins I and T are specific for cardiac muscle, and these biomarkers are currently recognized as reliable and critical markers of myocardial injury. An increase in myoglobin is not very specific in indicating an acute cardiac event; however, negative results are an excellent parameter for ruling out an acute MI.

A nurse is caring for a client who experienced an MI. The client is ordered to received metoprolol. The nurse understands that this medication has which therapeutic effect? Decreases resting heart rate Decreases platelet aggregation Decreases cholesterol level Increases cardiac output

Ans: Decreases resting heart rate Feedback: The therapeutic effects of beta-adrenergic blocking agents such as metoprolol are to reduce myocardial oxygen consumption by blocking beta-adrenergic sympathetic stimulation to the heart. The result is reduced heart rate, slowed conduction of impulses through the conduction system, decreased blood pressure, and reduced myocardial contractility to balance the myocardial oxygen needs and amount of oxygen available. This helps to control chest pain and delays the onset of ischemia during work or exercise. This classification of medication also reduces the incidence of recurrent angina, infarction, and cardiac mortality. In general, the dosage of medication is titrated to achieve a resting heart rate of 50-60 bpm. Metoprolol is not administered to decrease cholesterol levels, increase cardiac output, or decrease platelet aggregation.

Understanding atherosclerosis, the nurse identifies which of the following to be both a risk factor for the development of the disorder and an outcome? Hypertension Glucose intolerance Hyperlipidemia Obesity

Ans: Hypertension Feedback: Increases in diastolic and systolic blood pressure are associated with an increased incidence of atherosclerosis, often an inherited factor. Elevation of blood pressure results when the vessels cannot relax and impairs the ability of the artery to dilate. Hyperlipidemia, diabetes, and obesity are all risk factors for atherosclerosis but do not result from the disorder.

A client admitted to the coronary care unit (CCU) diagnosed with a STEMI is anxious and fearful. Which medication will the nurse administer to relieve the client's anxiety and decrease cardiac workload? Amlodipine IV morphine IV nitroglycerin Atenolol

Ans: IV morphine Feedback: IV morphine is the analgesic of choice for the treatment of an acute MI. It is given to reduce pain and treat anxiety. It also reduces preload and afterload, which decreases the workload of the heart. IV nitroglycerin is given to alleviate chest pain. Administration of atenolol and amlodipine are not indicated in this situation.

A client has been recently placed on nitroglycerin. Which instruction by the nurse should be included in the client's teaching plan? Instruct the client not to crush the tablet. Instruct the client to renew the nitroglycerin supply every 3 months. Instruct the client on side effects of flushing, throbbing headache, and tachycardia. Instruct the client to place nitroglycerin tablets in a plastic pill box.

Ans: Instruct the client on side effects of flushing, throbbing headache, and tachycardia. Feedback: The client should be instructed about side effects of the medication, which include flushing, throbbing headache, and tachycardia. The client should renew the nitroglycerin supply every 6 months. If the pain is severe, the client can crush the tablet between the teeth to hasten sublingual absorption. Tablets should never be removed and stored in metal or plastic pillboxes. Nitroglycerin is very unstable and should be carried in its original container.

Severe chest pain is reported by a client during an acute myocardial infarction. Which of the following is the most appropriate drug for the nurse to administer? Morphine sulfate (Morphine) Meperidine hydrochloride (Demerol) Isosorbide mononitrate (Isordil) Nitroglycerin transdermal patch

Ans: Morphine sulfate (Morphine) Feedback: Morphine sulfate not only decreases pain perception and anxiety but also helps to decrease heart rate, blood pressure, and demand for oxygen. Nitrates are administered for vasodilation and pain control in clients with angina-type pain, but oral forms (such as isosorbide dinitrate) have a large first-pass effect, and transdermal patch is used for long-term management. Meperidine hydrochloride is a synthetic opioid usually reserved for treatment of postoperative or migraine pain.

A nurse reviews a client's medication history before administering a cholinergic blocking agent. Adverse effects of a cholinergic blocking agent may delay absorption of what medication? Amantadine Nitroglycerin Digoxin Diphenhydramine

Ans: Nitroglycerin Feedback: A cholinergic blocking agent may cause dry mouth and delay the sublingual absorption of nitroglycerin. The nurse should offer the client sips of water before administering nitroglycerin. Amantadine, digoxin, and diphenhydramine can interact with a cholinergic blocking agent but not through delayed absorption. Amantadine and diphenhydramine enhance the effects of anticholinergic agents.

The nurse is teaching a client diagnosed with coronary artery disease about nitroglycerin. What is the cardiac premise behind administration of nitrates? Preload is reduced. It increases myocardial oxygen consumption. It functions has a vasoconstrictor. More blood returns to the heart.

Ans: Preload is reduced. Feedback: Nitroglycerin dilates primarily the veins, and in higher dosages, also the arteries. Dilation of the veins causes venous pooling of the blood throughout the body. As a result, less blood returns to the heart, and filling pressure (preload) is reduced. Nitroglycerine is administered to reduce myocardial oxygen consumption, which decreases ischemia and relieves pain.

Two female nursing assistants approach a nurse on a cardiac step-down unit to report that a client who experienced an acute myocardial infarction made sexual comments to them. How should the nurse intervene? The nurse should explain that the client most likely wants extra attention. The nurse should report the incident to her supervisor immediately. The nurse should explain that the client might have concerns about resuming sexual activity but is afraid to ask. The nurse should instruct the nursing assistants to avoid answering his call light.

Ans: The nurse should explain that the client might have concerns about resuming sexual activity but is afraid to ask. Feedback: Sometimes clients are concerned about resuming sexual activity but are afraid to ask. Making inappropriate sexual comments provides a forum for asking questions. It isn't necessary to report the incident to the nursing supervisor immediately without investigating the situation further. The client's call light must be answered in a timely fashion. More information is needed before assuming that the client is asking for extra attention.

Which term refers to preinfarction angina? Stable angina Variant angina Unstable angina Silent ischemia

Ans: Unstable angina Feedback: Preinfarction angina is also known as unstable angina. Stable angina has predictable and consistent pain that occurs upon exertion and is relieved by rest. Variant angina is exhibited by pain at rest and reversible ST-segment elevation. Silent angina manifests through evidence of ischemia, but the client reports no symptoms.

Which nursing actions would be of greatest importance in the management of a client preparing for angioplasty? Remove hair from skin insertion sites. Inform client of diagnostic tests. Assess distal pulses. Withhold anticoagulant therapy.

Ans: Withhold anticoagulant therapy. Feedback: The nurse knows to withhold the anticoagulant therapy to decrease chance of hemorrhage during the procedure. The nurse does inform the client of diagnostic test, will assess pulses, and prep the skin prior to the angioplasty, but this is not the most important action to be taken.

A client is admitted to the emergency department with chest pain and doesn't respond to nitroglycerin. The health care team obtains an electrocardiogram and administers I.V. morphine. The health care provider also considers administering alteplase. This thrombolytic agent must be administered how soon after onset of myocardial infarction (MI) symptoms? Within 6 hours Within 12 hours Within 24 to 48 hours Within 5 to 7 days

Ans: Within 6 hours Feedback: For the best chance of salvaging the client's myocardium, a thrombolytic agent must be administered within 6 hours after onset of chest pain or other signs or symptoms of MI. Sudden death is most likely to occur within the first 24 hours after an MI. Health care providers initiate I.V. heparin therapy after administration of a thrombolytic agent; it usually continues for 5 to 7 days.

Which client with a venous stasis ulcer is a candidate for topical hyperbaric oxygen therapy? a client whose ulcer includes necrotic tissue a nonambulatory client a client with a chronic, nonhealing skin lesion a client with an infected stasis ulcer

Ans: a client with a chronic, nonhealing skin lesion Feedback: Chronic, nonhealing skin lesions are treated with topical hyperbaric oxygen therapy. This approach delivers oxygen above atmospheric pressure directly to the wound rather than to the full body as with other disorders such as carbon monoxide poisoning. Necrotic tissue is debrided from a stasis ulcer. A client's infection is treated with an application of Silvadene, an antibacterial cream, or an antibiotic ointment and an occlusive transparent dressing such as Tegaderm that traps moisture and speeds healing.

The nurse is caring for a ventilated client after coronary artery bypass graft surgery. What are the criterions for extubation for the client? Select all that apply. adequate cough and gag reflexes inability to speak acceptable arterial blood gas values labile vital signs breathing without assistance of the ventilator

Ans: adequate cough and gag reflexes acceptable arterial blood gas values breathing without assistance of the ventilator Feedback: Before being extubated, the client should have cough and gag reflexes and stable vital signs; be able to lift the head off the bed or give firm hand grasps; have adequate vital capacity, negative inspiratory force, and minute volume appropriate for body size; and have acceptable arterial blood gas levels while breathing without the assistance of the ventilator. Inability to talk is expected when intubated with an endotracheal tube.

A client has had a 12-lead ECG completed as part of an annual physical examination. The nurse notes an abnormal Q wave on an otherwise unremarkable ECG. The nurse recognizes that this finding indicates an old MI. variant angina. an evolving MI. a cardiac dysrhythmia.

Ans: an old MI. Feedback: An abnormal Q wave may be present without ST-segment and T-wave changes, which indicates an old, not acute, MI.

The nurse recognizes that the treatment for a non-ST-elevation myocardial infarction (NSTEMI) differs from that for a STEMI, in that a STEMI is more frequently treated with percutaneous coronary intervention (PCI). IV heparin. IV nitroglycerin. thrombolytics.

Ans: percutaneous coronary intervention (PCI). Feedback: The client with a STEMI is often taken directly to the cardiac catheterization laboratory for an immediate PCI. Superior outcomes have been reported with the use of PCI compared to thrombolytics. IV heparin and IV nitroglycerin are used to treat NSTEMI.

A client presents to the ED with a myocardial infarction. Prior to administering a prescribed thrombolytic agent, the nurse must determine whether the client has which absolute contraindication to thrombolytic therapy? shellfish allergy prior intracranial hemorrhage use of heparin recent consumption of a meal

Ans: prior intracranial hemorrhage Feedback: History of a prior intracranial hemorrhage is an absolute contraindication for thrombolytic therapy. An allergy to iodine, shellfish, radiographic dye, and latex are of primary concern before a cardiac catheterization but not a known contraindication for thrombolytic therapy. Administration of a thrombolytic agent with heparin increases risk of bleeding; the primary healthcare provider usually discontinues the heparin until thrombolytic treatment is completed.

A client was transferring a load of firewood in the morning and experienced a heaviness in the chest and dyspnea. The client arrives in the emergency department four hours after the heaviness and the health care provider diagnoses an anterior myocardial infarction (MI). What orders will the nurse anticipate? streptokinase, aspirin, and morphine administration serial liver enzyme testing, telemetry, and a lidocaine infusion sublingual nitroglycerin, tissue plasminogen activator (tPA), and telemetry morphine administration, stress testing, and admission to the cardiac care unit

Ans: sublingual nitroglycerin, tissue plasminogen activator (tPA), and telemetry Feedback: The nurse should anticipate an order for sublingual nitroglycerin, tPA, and telemetry, as the client's chest pain began 4 hours before diagnosis. The preferred choice is tPA, which is more specific for cardiac tissue than streptokinase. Stress testing shouldn't be performed during an MI. The client doesn't exhibit symptoms that indicate the use of lidocaine.

A client who is resting quietly reports chest pain to the nurse. The cardiac monitor indicates the presence of reversible ST-segment elevation. What type of angina is the client experiencing? stable angina silent angina variant angina intractable angina

Ans: variant angina Feedback: Variant or Prinzmetal's angina is distinguished by pain occurrence during rest. Stable angina occurs with activity. Silent angina occurs without symptoms, and intractable angina is evidenced by incapacitating pain.

A client who had coronary artery bypass surgery is exhibiting signs of heart failure. What medications will the nurse anticipate administering for this client? Select all that apply. a) digoxin b) nitroprusside c) diuretics d) amlodipine e) inotropic agents

Ans: diuretics inotropic agents digoxin Feedback: Medical management of cardiac failure includes digoxin, diuretics, and IV inotropic agents. Amlodipine and calcium channel blockers are not used due to systolic dysfunction. Nitroprusside is a vasodilator that is not used for heart failure.

You are writing a care plan for a patient who has been diagnosed with angina pectoris. The patient describes herself as being "distressed" and "shocked" by her new diagnosis. What nursing diagnosis is most clearly suggested by the woman's statement? A) Spiritual distress related to change in health status B) Acute confusion related to prognosis for recovery C) Anxiety related to cardiac symptoms D) Deficient knowledge related to treatment of angina pectoris

Ans: Anxiety related to cardiac symptoms Feedback: Although further assessment is warranted, it is not unlikely that the patient is experiencing anxiety. In patients with CAD, this often relates to the threat of sudden death. There is no evidence of confusion (i.e., delirium or dementia) and there may or may not be a spiritual element to her concerns. Similarly, it is not clear that a lack of knowledge or information is the root of her anxiety.

The nurse is assessing a client with suspected postpericardiotomy syndrome after cardiac surgery. What manifestation will alert the nurse to this syndrome? hypothermia decreased erythrocyte sedimentation rate (ESR) pericardial friction rub decreased white blood cell (WBC) count

Ans: pericardial friction rub Feedback: Postpericardiotomy syndrome is characterized by fever, pericardial pain, pleural pain, dyspnea, pericardial effusion, pericardial friction rub, and arthralgia. Leukocytosis (elevated WBCs) occurs, along with elevation of the ESR. Hypothermia is not a symptom of postpericardiotomy syndrome.

The nurse has completed a teaching session on self-administration of sublingual nitroglycerin. Which client statement indicates that the teaching has been effective? "I can put the nitroglycerin tablets in my daily pill dispenser with my other medications". "Side effects of nitroglycerin include flushing, throbbing headache, and hypertension". "I can take nitroglycerin before sex so I won't develop chest pain". "After taking two tablets with no relief, I should call EMS."

Ans: "I can take nitroglycerin before sex so I won't develop chest pain". Feedback: Nitroglycerin can be taken in anticipation of any activity that may produce pain. Because nitroglycerin increases tolerance for exercise and stress when taken prophylactically (i.e., before an angina-producing activity such as exercise, stair-climbing, or sexual intercourse), it is best taken before pain develops. The client is instructed to take three tablets 5 minutes apart; if the chest pain is not relieved, emergency medical services should be contacted. Nitroglycerin is very unstable; it should be carried securely in its original container (e.g., a capped dark glass bottle), and tablets should never be removed and stored in metal or plastic pillboxes. Side effects of nitroglycerin include flushing, throbbing headache, hypotension, and tachycardia.

A client is ordered a nitroglycerine transdermal patch for treatment of CAD and asks the nurse why the patch is removed at bedtime. Which is the best response by the nurse? "Nitroglycerine causes headaches, but removing the patch decreases the incidence." "You do not need the effects of nitroglycerine while you sleep." "Removing the patch at night prevents drug tolerance while keeping the benefits." "Contact dermatitis and skin irritations are common when the patch remains on all day."

Ans: "Removing the patch at night prevents drug tolerance while keeping the benefits." Feedback: Tolerance to antiangina effects of nitrates can occur when taking these drugs for long periods of time. Therefore, to prevent tolerance and maintain benefits, it is a common regime to remove transdermal patches at night. Common adverse effects of nitroglycerin are headaches and contact dermatitis but not the reason for removing the patch at night. It is true that while the client rests, there is less demand on the heart but not the primary reason for removing the patch.

The nurse is teaching the client about coronary artery damage after an abnormal fasting lipid profile. The client asks the nurse what type of lipid is most troublesome. What is the nurse's best response? "The total cholesterol level of 252 mg/dL warrants medication treatment alone." "The low-density lipoproteins (LDL) pose a threat to plague formation and can cause a heart attack of stroke." "The higher the high-density lipoproteins (HDL), the more at risk you are for heart damage or a stroke." "The triglycerides levels measure good fat, so the higher the level, the less risk you are for a heart attack or stroke."

Ans: "The low-density lipoproteins (LDL) pose a threat to plague formation and can cause a heart attack of stroke." Feedback: When there is an excess of LDL, these particles adhere to vulnerable points in the arterial endothelium. Here, macrophages ingest them, leading to the formation of foam cells and the beginning of plaque formation. A harmful effect is exerted on the coronary vasculature because the small LDL particles can be easily transported into the vessel lining. The cholesterol level should be <200 mg/dL but it is not the only indication for treatment. The lower the HDL, the more the client is at risk for heart attack or stroke. The combination of the client's triglycerides, LDL, and HDL levels is used to direct treatment.

A client asks the clinic nurse what the difference is between arteriosclerosis and atherosclerosis. What is the nurse's best response? Arteriosclerosis is a loss of elasticity, or hardening of the arteries, that happens as we age. Atherosclerosis is a loss of elasticity, or hardening of the arteries, that happens as we age. Arteriosclerosis is a formation of clots in the inner lining of the arteries. Atherosclerosis is a formation of clots in the inner lining of the arteries.

Ans: Arteriosclerosis is a loss of elasticity, or hardening of the arteries, that happens as we age. Feedback: Arteriosclerosis refers to the loss of elasticity or hardening of the arteries, that accompanies the aging process. Therefore, options B, C, and D are incorrect.

When starting a client on oral or I.V. diltiazem, for which potential complication should the nurse monitor? Atrioventricular block Renal failure Hypertension Flushing

Ans: Atrioventricular block Feedback: The chief complications of diltiazem are hypotension, atrioventricular blocks, heart failure, and elevated liver enzyme levels. Other reported reactions include flushing, nocturia, and polyuria, but not renal failure. Although flushing may occur, it's an adverse reaction, not a potential complication.

The nurse is caring for a client with Raynaud's disease. What is an important instruction for a client who is diagnosed with this disease to prevent an attack? Avoid fatty foods and exercise. Take over-the-counter decongestants. Avoid situations that contribute to ischemic episodes. Report changes in the usual pattern of chest pain.

Ans: Avoid situations that contribute to ischemic episodes. Feedback: Teaching for clients with Raynaud's disease and their family members is important. They need to understand what contributes to an attack. The nurse should instruct the clients to avoid situations that contribute to ischemic episodes. Reporting changes in the usual pattern of chest pain or avoiding fatty foods and exercise does not help the client to avoid an attack; it is more contributory for clients with CAD. In addition, the nurse advises clients to avoid over-the-counter decongestants.

The nurse is caring for a client with Raynaud's disease. What is an important instruction for a client who is diagnosed with this disease to prevent an attack? Report changes in the usual pattern of chest pain. Avoid situations that contribute to ischemic episodes. Avoid fatty foods and exercise. Take over-the-counter decongestants.

Ans: Avoid situations that contribute to ischemic episodes. Feedback: Teaching for clients with Raynaud's disease and their family members is important. They need to understand what contributes to an attack. The nurse should instruct the clients to avoid situations that contribute to ischemic episodes. Reporting changes in the usual pattern of chest pain or avoiding fatty foods and exercise does not help the client to avoid an attack; it is more contributory for clients with CAD. In addition, the nurse advises clients to avoid over-the-counter decongestants.

A client reports chest pain and palpitations during and after jogging in the mornings. The client's family history reveals a history of coronary artery disease (CAD). What should the nurse recommend to minimize cardiac risk? Protein-rich diet Liquid diet Smoking cessation Mild meals

Ans: Smoking cessation Feedback: The first line of defense for clients with CAD is a change in lifestyle, such as smoking cessation, weight loss, stress management, and exercise. A protein-rich diet, liquid diet, and mild meals will not minimize cardiac risk.

A nurse is caring for a client in the cardiovascular intensive care unit following a coronary artery bypass graft. Which clinical finding requires immediate intervention by the nurse? Heart rate 66 bpm Blood pressure 110/68 mm Hg Central venous pressure reading of 1 Pain score 5/10

Ans: Central venous pressure reading of 1 Feedback: The central venous pressure (CVP) reading of 1 is low (2-6 mm Hg) and indicates reduced right ventricular preload, commonly caused by hypovolemia. Hypovolemia is the most common cause of decreased cardiac output after cardiac surgery. Replacement fluids such as colloids, packed red blood cells, or crystalloid solutions may be prescribed. The other findings require follow-up by the nurse; however, addressing the CVP reading is the nurse's priority.

A middle-aged client presents to the ED reporting severe chest discomfort. Which finding is most indicative of a possible myocardial infarction (MI)? Chest discomfort not relieved by rest or nitroglycerin Intermittent nausea and emesis for 3 days Cool, clammy skin and a diaphoretic, pale appearance Anxiousness, restlessness, and lightheadedness

Ans: Chest discomfort not relieved by rest or nitroglycerin Feedback: Chest pain or discomfort not relieved by rest or nitroglycerin is associated with an acute MI. The other findings, although associated with acute coronary syndrome or MI, may also occur with angina and, alone, are not indicative of an MI.

Which medication is given to clients who are diagnosed with angina but are allergic to aspirin? Clopidogrel Amlodipine Diltiazem Felodipine

Ans: Clopidogrel Feedback: Clopidogrel or ticlopidine is given to clients who are allergic to aspirin or are given in addition to aspirin to clients who are at high risk for MI. Amlodipine, diltiazem, and felodipine are calcium channel blockers.

A client is given a prescription for metoprolol after being examined by the health care provider. What is the most important teaching for the nurse to give to the client? If dizziness occurs, adjust the medication. Dress warmly. Blood circulation may be reduced in the extremities. Take the medication at the same time each day. Don't suddenly stop taking the medication without calling your health care provider.

Ans: Don't suddenly stop taking the medication without calling your health care provider. Feedback: All teaching points need to be covered, but the nurse needs to emphasize that metoprolol should not be suddenly stopped because some conditions can become worse.

A nurse is educating a community group about coronary artery disease. One member asks about how to avoid coronary artery disease. Which of the following items are considered modifiable risk factors for coronary artery disease? Choose all that apply. Obesity Gender Race Tobacco use Hyperlipidemia

Ans: Hyperlipidemia Obesity Tobacco use Feedback: Modifiable risk factors for coronary artery disease include hyperlipidemia, tobacco use, hypertension, diabetes mellitus, metabolic syndrome, obesity, and physical inactivity. Nonmodifiable risk factors include family history, advanced age, gender, and race.

The nurse notes that the post cardiac surgery client demonstrates low urine output (< 25 mL/hr) with high specific gravity (> 1.025). What will the nurse anticipate the health care provider will order? Increase intravenous fluids Decrease intravenous fluids Irrigate the urinary catheter Prepare the client for dialysis

Ans: Increase intravenous fluids Feedback: Urine output of less than 25 mL/hr may indicate a decrease in cardiac output. A high specific gravity indicates increased concentration of solutes in the urine, which occurs with inadequate fluid volume. The healthcare provider may increase intravenous fluids. Irrigating the urinary catheter will be done if there is a suspected blockage. Dialysis is not indicated by urinary volumes.

A nurse is teaching a client who receives nitrates for the relief of chest pain. Which instruction should the nurse emphasize? Repeat the dose of sublingual nitroglycerin every 15 minutes for three doses. Store the drug in a cool, well-lit place. Lie down or sit in a chair for 5 to 10 minutes after taking the drug. Restrict alcohol intake to two drinks per day.

Ans: Lie down or sit in a chair for 5 to 10 minutes after taking the drug. Feedback: Nitrates act primarily to relax coronary smooth muscle and produce vasodilation. They can cause hypotension, which makes the client dizzy and weak. The nurse should instruct the client to lie down or sit in a chair for 5 to 10 minutes after taking the drug. Nitrates are taken at the first sign of chest pain and before activities that might induce chest pain. Sublingual nitroglycerin is taken every 5 minutes for three doses. If the pain persists, the client should seek medical assistance immediately. Nitrates must be stored in a dark place in a closed container because sunlight causes the medication to lose its effectiveness. Alcohol is prohibited because nitrates may enhance the effects of the alcohol.

A patient in the recovery room after cardiac surgery begins to have extremity paresthesia, peaked T waves, and mental confusion. What type of electrolyte imbalance does the nurse suspect this patient is having? Calcium Magnesium Potassium Sodium

Ans: Potassium Feedback: Hyperkalemia (high potassium) can result in the following ECG changes: tall peaked T waves, wide QRS, and bradycardia. The nurse should be prepared to administer a diuretic or an ion-exchange resin (sodium polystyrene sulfonate [Kayexalate]); IV sodium bicarbonate, or IV insulin and glucose. Imbalances in the other electrolytes listed would not result in peaked T waves.

The nurse is caring for a client following a coronary artery bypass graft (CABG). The nurse notes persistent oozing of bloody drainage from various puncture sites. The nurse anticipates that the physician will order which medication to neutralize the unfractionated heparin the client received? Protamine sulfate Alteplase Clopidogrel Aspirin

Ans: Protamine sulfate Feedback: Protamine sulfate is known as the antagonist for unfractionated heparin (it neutralizes heparin). Alteplase is a thrombolytic agent. Clopidogrel is an antiplatelet medication that is given to reduce the risk of thrombus formation after coronary stent placement. The antiplatelet effect of aspirin does not reverse the effects of heparin.

Which medication is an antidote to heparin? Alteplase Clopidogrel Protamine sulfate Aspirin

Ans: Protamine sulfate Feedback: Protamine sulfate is known as the antagonist to heparin. Alteplase is a thrombolytic agent. Clopidogrel is an antiplatelet medication that is given to reduce the risk of thrombus formation post coronary stent placement. The antiplatelet effect of aspirin does not reverse the effects of heparin.

A client is receiving morphine to relieve chest pain. The order is for 4 mg IV now. The pharmacy supplies morphine sulfate at 5 mg per mL. How many mL will the nurse give the client? Enter the correct number ONLY. _____ mL

Ans: 0.8 mL Feedback: (4 mg/5 mg) X 1 mL = 0.8 mL

The client is prescribed nadolol for hypertension. What is the reason the nurse will teach the client not to stop taking the medication abruptly? The abrupt stop can cause a myocardial infarction. The abrupt stop can lead to formation of blood clots. The abrupt stop will precipitate internal bleeding. The abrupt stop can trigger a migraine headache.

Ans: The abrupt stop can cause a myocardial infarction. Feedback: Patients taking beta blockers are cautioned not to stop taking them abruptly because angina may worsen and myocardial infarction may develop. Beta blockers do not cause the formation of blood clots, internal bleeding, or the onset of a migraine headache.

A patient with coronary artery disease (CAD) is having a cardiac catheterization. What indicator is present for the patient to have a coronary artery bypass graft (CABG)? The patient has compromised left ventricular function. The patient has had angina longer than 3 years. The patient has at least a 70% occlusion of a major coronary artery. The patient has an ejection fraction of 65%.

Ans: The patient has at least a 70% occlusion of a major coronary artery. Feedback: For a patient to be considered for CABG, the coronary arteries to be bypassed must have approximately a 70% occlusion (60% if in the left main coronary artery).

The nurse is teaching a client about atherosclerosis. The client asks the nurse what the substance causing atherosclerosis is made of. How does the nurse best respond? fatty deposits in the lumen of arteries cholesterol plugs in the lumen of veins blood clots in the arteries emboli in the veins

Ans: fatty deposits in the lumen of arteries Feedback: Atherosclerosis is a condition in which the lumen of arteries fill with fatty deposits called plaque. The plaque does not involve blood clots in arteries, emboli in veins or cholesterol plugs in veins.

A client with an acute myocardial infarction is receiving nitroglycerin by continuous I.V. infusion. Which client statement indicates that this drug is producing its therapeutic effect? "My chest pain is decreasing." "I have a bad headache." "I feel a tingling sensation around my mouth." "My vision is blurred, so my blood pressure must be up."

Ans: "My chest pain is decreasing." Explanation: Nitroglycerin, a vasodilator, increases the arterial supply of oxygen-rich blood to the myocardium. This action produces the drug's intended effect: relief of chest pain. Headache is an adverse effect of nitroglycerin. The drug shouldn't cause a tingling sensation around the mouth and should lower, not raise, blood pressure.

A client is ordered a nitroglycerine transdermal patch for treatment of CAD and asks the nurse why the patch is removed at bedtime. Which is the best response by the nurse? "Nitroglycerine causes headaches, but removing the patch decreases the incidence." "You do not need the effects of nitroglycerine while you sleep." "Contact dermatitis and skin irritations are common when the patch remains on all day." "Removing the patch at night prevents drug tolerance while keeping the benefits."

Ans: "Removing the patch at night prevents drug tolerance while keeping the benefits." Explanation: Tolerance to antiangina effects of nitrates can occur when taking these drugs for long periods of time. Therefore, to prevent tolerance and maintain benefits, it is a common regime to remove transdermal patches at night. Common adverse effects of nitroglycerin are headaches and contact dermatitis but not the reason for removing the patch at night. It is true that while the client rests, there is less demand on the heart but not the primary reason for removing the patch.

The nurse providing care for a patient post PTCA knows to monitor the patient closely. For what complications should the nurse monitor the patient? Select all that apply. A) Abrupt closure of the coronary artery B) Venous insufficiency C) Bleeding at the insertion site D) Retroperitoneal bleeding E) Arterial occlusion

Ans: - Abrupt closure of the coronary artery - Bleeding at the insertion site - Retroperitoneal bleeding - Arterial occlusion Feedback: Complications after the procedure may include abrupt closure of the coronary artery and vascular complications, such as bleeding at the insertion site, retroperitoneal bleeding, hematoma, and arterial occlusion, as well as acute renal failure. Venous insufficiency is not a postprocedure complication of a PTCA.

The client is asking the nurse about heart-healthy food choices for lunch. What are foods that are heart healthy? Select all that apply. white rice with butter soy yogurt blueberries baked chicken leg broiled trout

Ans: soy yogurt broiled trout blueberries Feedback: Heart-healthy foods include soy products, fish high in omega-3s, and fruit. The chicken leg has more fat than a chicken breast. The white rice does not have enough fiber, so brown rice is a better option. (NO DARK MEAT CHICKEN)

A client with known coronary artery disease reports intermittent chest pain, usually on exertion. When teaching the client about nitroglycerin administration, which instruction should the nurse provide? A) "You may take a sublingual nitroglycerin tablet every 30 minutes, if needed. You may take as many as four doses." B) "Replace leftover sublingual nitroglycerin tablets every 9 months to make sure your pills are fresh." C) "Be sure to take safety precautions because nitroglycerin may cause dizziness when you stand up." D) "A burning sensation after administration indicates that the nitroglycerin tablets are potent."

Ans: "Be sure to take safety precautions because nitroglycerin may cause dizziness when you stand up." Feedback: Nitroglycerin commonly causes orthostatic hypotension and dizziness. To minimize these problems, the nurse should teach the client to take safety precautions, such as changing to an upright position slowly, climbing up and down stairs carefully, and lying down at the first sign of dizziness. To ensure the freshness of sublingual nitroglycerin, the client should replace tablets every 6 months, not every 9 months, and store them in a tightly closed container in a cool, dark place. Many brands of sublingual nitroglycerin no longer produce a burning sensation. The client should take a sublingual nitroglycerin tablet at the first sign of angina. He may repeat the dose every 5 minutes for up to three doses; if this intervention doesn't bring relief, the client should seek immediate medical attention.

In preparation for cardiac surgery, a patient was taught about measures to prevent venous thromboembolism. What statement indicates that the patient clearly understood this education? A) "I'll try to stay in bed for the first few days to allow myself to heal." B) "I'll make sure that I don't cross my legs when I'm resting in bed." C) "I'll keep pillows under my knees to help my blood circulate better." D) "I'll put on those compression stockings if I get pain in my calves."

Ans: "I'll make sure that I don't cross my legs when I'm resting in bed." Feedback: To prevent venous thromboembolism, patients should avoid crossing the legs. Activity is generally begun as soon as possible and pillows should not be placed under the popliteal space. Compression stockings are often used to prevent venous thromboembolism, but they would not be applied when symptoms emerge.

A client presents to the emergency department reporting chest pain. Which order should the nurse complete first? a) Aspirin 325 mg orally b) Troponin level c) 12-lead ECG d) 2 L oxygen via nasal cannula

Ans: 12-lead ECG Feedback: The nurse should complete the 12-lead ECG first. The priority is to determine whether the client is suffering an acute MI and implement appropriate interventions as quickly as possible. The other orders should be completed after the ECG.

A nurse is working with a patient who has been scheduled for a percutaneous coronary intervention (PCI) later in the week. What anticipatory guidance should the nurse provide to the patient? A) He will remain on bed rest for 48 to 72 hours after the procedure. B) He will be given vitamin K infusions to prevent bleeding following PCI. C) A sheath will be placed over the insertion site after the procedure is finished. D) The procedure will likely be repeated in 6 to 8 weeks to ensure success.

Ans: A sheath will be placed over the insertion site after the procedure is finished. Feedback: A sheath is placed over the PCI access site and kept in place until adequate coagulation is achieved. Patients resume activity a few hours after PCI and repeated treatments may or may not be necessary. Anticoagulants, not vitamin K, are administered during PCI.

The nurse is creating a plan of care for a patient with acute coronary syndrome. What nursing action should be included in the patient's care plan? A) Facilitate daily arterial blood gas (ABG) sampling. B) Administer supplementary oxygen, as needed. C) Have patient maintain supine positioning when in bed. D) Perform chest physiotherapy, as indicated.

Ans: Administer supplementary oxygen, as needed. Feedback: Oxygen should be administered along with medication therapy to assist with symptom relief. Administration of oxygen raises the circulating level of oxygen to reduce pain associated with low levels of myocardial oxygen. Physical rest in bed with the head of the bed elevated or in a supportive chair helps decrease chest discomfort and dyspnea. ABGs are diagnostic, not therapeutic, and they are rarely needed on a daily basis. Chest physiotherapy is not used in the treatment of ACS.

A client with chronic arterial occlusive disease undergoes percutaneous transluminal coronary angioplasty (PTCA) for mechanical dilation of the right femoral artery. After the procedure, the client will be prescribed long-term administration of which drug? penicillin V or erythromycin. pentoxifylline or acetaminophen. aspirin or acetaminophen. aspirin or clopidogrel.

Ans: Aspirin or clopidogrel. Feedback: After PTCA, the client begins long-term aspirin or clopidogrel therapy to prevent thromboembolism. Health care providers order heparin for anticoagulation during this procedure; some health care providers discharge clients with a prescription for long-term warfarin or low-molecular-weight heparin therapy. Pentoxifylline, a vasodilator used to treat chronic arterial occlusion, isn't required after PTCA because the procedure itself opens the vessel. The health care provider may order short-term acetaminophen therapy to manage fever or discomfort, but prolonged therapy isn't warranted. The client may need an antibiotic, such as penicillin or erythromycin, for a brief period to prevent infection associated with an invasive procedure; long-term therapy isn't necessary.

A client reports chest pain and heavy breathing when exercising or when stressed. Which is a priority nursing intervention for the client diagnosed with coronary artery disease? A) Assess chest pain and administer prescribed drugs and oxygen B) It is not important to assess the client or to notify the physician C) Assess blood pressure and administer aspirin D) Assess the client's physical history

Ans: Assess chest pain and administer prescribed drugs and oxygen Feedback: The nurse assesses the client for chest pain and administers the prescribed drugs that dilate the coronary arteries. The nurse administers oxygen to improve the oxygen supply to the heart. Assessing blood pressure or the client's physical history does not clearly indicate that the client has CAD. The nurse does not administer aspirin without a prescription from the physician.

The nurse is participating in the care conference for a patient with ACS. What goal should guide the care team's selection of assessments, interventions, and treatments? A) Maximizing cardiac output while minimizing heart rate B) Decreasing energy expenditure of the myocardium C) Balancing myocardial oxygen supply with demand D) Increasing the size of the myocardial muscle

Ans: Balancing myocardial oxygen supply with demand Feedback: Balancing myocardial oxygen supply with demand (e.g., as evidenced by the relief of chest pain) is the top priority in the care of the patient with ACS. Treatment is not aimed directly at minimizing heart rate because some patients experience bradycardia. Increasing the size of the myocardium is never a goal. Reducing the myocardium's energy expenditure is often beneficial, but this must be balanced with productivity.

A 48-year-old man presents to the ED complaining of severe substernal chest pain radiating down his left arm. He is admitted to the coronary care unit (CCU) with a diagnosis of myocardial infarction (MI). What nursing assessment activity is a priority on admission to the CCU? A) Begin ECG monitoring. B) Obtain information about family history of heart disease. C) Auscultate lung fields. D) Determine if the patient smokes.

Ans: Begin ECG monitoring. Feedback: The 12-lead ECG provides information that assists in ruling out or diagnosing an acute MI. It should be obtained within 10 minutes from the time a patient reports pain or arrives in the ED. By monitoring serial ECG changes over time, the location, evolution, and resolution of an MI can be identified and monitored; life-threatening arrhythmias are the leading cause of death in the first hours after an MI. Obtaining information about family history of heart disease and whether the patient smokes are not immediate priorities in the acute phase of MI. Data may be obtained from family members later. Lung fields are auscultated after oxygenation and pain control needs are met.

The nurse working on the coronary care unit is caring for a patient with ACS. How can the nurse best meet the patient's psychosocial needs? A) Reinforce the fact that treatment will be successful. B) Facilitate a referral to a chaplain or spiritual leader. C) Increase the patient's participation in rehabilitation activities. D) Directly address the patient's anxieties and fears.

Ans: Directly address the patient's anxieties and fears. Feedback: Alleviating anxiety and decreasing fear are important nursing functions that reduce the sympathetic stress response. Referrals to spiritual care may or may not be appropriate, and this does not relieve the nurse of responsibility for addressing the patient's psychosocial needs. Treatment is not always successful, and false hope should never be fostered. Participation in rehabilitation may alleviate anxiety for some patients, but it may exacerbate it for others

A patient with an occluded coronary artery is admitted and has an emergency percutaneous transluminal coronary angioplasty (PTCA). The patient is admitted to the cardiac critical care unit after the PTCA. For what complication should the nurse most closely monitor the patient? A) Hyperlipidemia B) Bleeding at insertion site C) Left ventricular hypertrophy D) Congestive heart failure ____________________________________________________ A client with chronic arterial occlusive disease undergoes percutaneous transluminal coronary angioplasty (PTCA) for mechanical dilation of the right femoral artery. After the procedure, the client will be prescribed long-term administration of which drug? penicillin V or erythromycin. pentoxifylline or acetaminophen. aspirin or acetaminophen. aspirin or clopidogrel.

Ans: Bleeding at insertion site Feedback: Complications of PTCA may include bleeding at the insertion site, abrupt closure of the artery, arterial thrombosis, and perforation of the artery. Complications do not include hyperlipidemia, left ventricular hypertrophy, or congestive heart failure; each of these problems takes an extended time to develop and none is emergent. ____________________________________________________ Ans: Aspirin or clopidogrel. Feedback: After PTCA, the client begins long-term aspirin or clopidogrel therapy to prevent thromboembolism. Health care providers order heparin for anticoagulation during this procedure; some health care providers discharge clients with a prescription for long-term warfarin or low-molecular-weight heparin therapy. Pentoxifylline, a vasodilator used to treat chronic arterial occlusion, isn't required after PTCA because the procedure itself opens the vessel. The health care provider may order short-term acetaminophen therapy to manage fever or discomfort, but prolonged therapy isn't warranted. The client may need an antibiotic, such as penicillin or erythromycin, for a brief period to prevent infection associated with an invasive procedure; long-term therapy isn't necessary.

A patient in the cardiac step-down unit has begun bleeding from the percutaneous coronary intervention (PCI) access site in her femoral region. What is the nurse's most appropriate action? A) Call for assistance and initiate cardiopulmonary resuscitation. B) Reposition the patient's leg in a nondependent position. C) Promptly remove the femoral sheath. D) Call for help and apply pressure to the access site.

Ans: Call for help and apply pressure to the access site. Feedback: The femoral sheath produces pressure on the access site. Pressure will temporarily reduce bleeding and allow for subsequent interventions. Removing the sheath would exacerbate bleeding and repositioning would not halt it. CPR is not indicated unless there is evidence of respiratory or cardiac arrest.

Which complication of cardiac surgery occurs when there is fluid and clot accumulation in the pericardial sac, which compresses the heart, preventing blood from filling the ventricles? a) Hypertension b) Cardiac tamponade c) Hypothermia d) Fluid overload

Ans: Cardiac tamponade Feedback: Cardiac tamponade is fluid and clot accumulation in the pericardial sac, which compresses the heart, preventing the blood from filling the ventricles. Fluid overload is exhibited by high pulmonary artery wedge pressure, central venous pressure, and pulmonary artery diastolic pressure, as well as crackles in the lungs. Hypertension results from postoperative vasoconstriction. Hypothermia is a low body temperature that leads to vasoconstriction.

The OR nurse is explaining to a patient that cardiac surgery requires the absence of blood from the surgical field. At the same time, it is imperative to maintain perfusion of body organs and tissues. What technique for achieving these simultaneous goals should the nurse describe? A) Coronary artery bypass graft (CABG) B) Percutaneous transluminal coronary angioplasty (PTCA) C) Atherectomy D) Cardiopulmonary bypass

Ans: Cardiopulmonary bypass Feedback: Cardiopulmonary bypass is often used to circulate and oxygenate blood mechanically while bypassing the heart and lungs. PTCA, atherectomy, and CABG are all surgical procedures, none of which achieves the two goals listed.

A nurse has taken on the care of a patient who had a coronary artery stent placed yesterday. When reviewing the patient's daily medication administration record, the nurse should anticipate administering what drug? A) Ibuprofen B) Clopidogrel C) Dipyridamole D) Acetaminophen

Ans: Clopidogrel Feedback: Because of the risk of thrombus formation within the stent, the patient receives antiplatelet medications, usually aspirin and clopidogrel. Ibuprofen and acetaminophen are not antiplatelet drugs. Dipyridamole is not the drug of choice following stent placement.

A patient who is postoperative day 1 following a CABG has produced 20 mL of urine in the past 3 hours and the nurse has confirmed the patency of the urinary catheter. What is the nurse's most appropriate action? A) Document the patient's low urine output and monitor closely for the next several hours. B) Contact the dietitian and suggest the need for increased oral fluid intake. C) Contact the patient's physician and suggest assessment of fluid balance and renal function. D) Increase the infusion rate of the patient's IV fluid to prompt an increase in renal function.

Ans: Contact the patient's physician and suggest assessment of fluid balance and renal function. Feedback: Nursing management includes accurate measurement of urine output. An output of less than 1 mL/kg/h may indicate hypovolemia or renal insufficiency. Prompt referral is necessary. IV fluid replacement may be indicated, but is beyond the independent scope of the dietitian or nurse.

A patient presents to the walk-in clinic complaining of intermittent chest pain on exertion, which is eventually attributed to angina. The nurse should inform the patient that angina is most often attributable to what cause? A) Decreased cardiac output B) Decreased cardiac contractility C) Infarction of the myocardium D) Coronary arteriosclerosis

Ans: Coronary arteriosclerosis Feedback: In most cases, angina pectoris is due to arteriosclerosis. The disease is not a result of impaired cardiac output or contractility. Infarction may result from untreated angina, but it is not a cause of the disease.

The public health nurse is participating in a health fair and interviews a patient with a history of hypertension, who is currently smoking one pack of cigarettes per day. She denies any of the most common manifestations of CAD. Based on these data, the nurse would expect the focuses of CAD treatment most likely to be which of the following? A) Drug therapy and smoking cessation B) Diet and drug therapy C) Diet therapy only D) Diet therapy and smoking cessation

Ans: Diet therapy and smoking cessation Feedback: Due to the absence of symptoms, dietary therapy would likely be selected as the first-line treatment for possible CAD. Drug therapy would be determined based on a number of considerations and diagnostics findings, but would not be directly indicated. Smoking cessation is always indicated, regardless of the presence or absence of symptoms.

The nurse is assessing a patient with acute coronary syndrome (ACS). The nurse includes a careful history in the assessment, especially with regard to signs and symptoms. What signs and symptoms are suggestive of ACS? Select all that apply. A) Dyspnea B) Unusual fatigue C) Hypotension D) Syncope E) Peripheral cyanosis

Ans: Dyspnea, Unusual fatigue, Syncope Feedback: Systematic assessment includes a careful history, particularly as it relates to symptoms: chest pain or discomfort, difficulty breathing (dyspnea), palpitations, unusual fatigue, faintness (syncope), or sweating (diaphoresis). Each symptom must be evaluated with regard to time, duration, and the factors that precipitate the symptom and relieve it, and in comparison with previous symptoms. Hypotension and peripheral cyanosis are not typically associated with ACS.

A client comes to the emergency department reporting chest pain. An electrocardiogram (ECG) reveals myocardial ischemia and an anterior-wall myocardial infarction (MI). Which ECG characteristic does the nurse expect to see? A) Elevated ST segment B) Absent Q wave C) Prolonged PR interval D) Widened QRS complex

Ans: Elevated ST segment Feedback: Ischemic myocardial tissue changes cause elevation of the ST segment, an inverted T wave, and a pathological Q wave. A prolonged PR interval occurs with first-degree heart block, the least dangerous atrioventricular heart block; this disorder may arise in healthy people but sometimes results from drug toxicity, electrolyte or metabolic disturbances, rheumatic fever, or chronic degenerative disease of the conduction system. An absent Q wave is normal; an MI may cause a significant Q wave. A widened QRS complex indicates a conduction delay in the His-Purkinje system. The first signs of an acute MI are usually seen in the T wave and the ST segment. The T wave becomes inverted; the ST segment elevates (it is usually flat). An elevated ST segment in two contiguous leads is a key diagnostic indicator for MI (i.e., ST-elevation MI). This client requires immediate invasive therapy or fibrinolytic medications. Although the other ECG findings require intervention, elevated ST elevations require immediate and definitive interventions.

A female client returns for a follow-up visit to the cardiologist 4 days after a trip to the ED for sudden shortness of breath and abdominal pain. The nurse realizes the client had a myocardial infarction because the results from the blood work drawn in the hospital shows: a) Elevated troponin levels b) Increased C-reactive protein levels c) Decreased LDH levels d) Decreased myoglobin levels

Ans: Elevated troponin levels Feedback: Troponin is present only in myocardial tissue; therefore, it is the gold standard for determining heart damage in the early stages of an MI. LDH1 and LDH2 may be elevated in response to cardiac or other organ damage during an MI. Myoglobin is a biomarker that rises in 2 to 3 hours after heart damage during an MI. C-reactive protein, erythrocyte sedimentation rate, and the WBC count increase on about the third day following MI because of the inflammatory response that the injured myocardial cells triggered. These levels would not be elevated during the MI event.

When discussing angina pectoris secondary to atherosclerotic disease with a patient, the patient asks why he tends to experience chest pain when he exerts himself. The nurse should describe which of the following phenomena? A) Exercise increases the heart's oxygen demands. B) Exercise causes vasoconstriction of the coronary arteries. C) Exercise shunts blood flow from the heart to the mesenteric area. D) Exercise increases the metabolism of cardiac medications.

Ans: Exercise increases the heart's oxygen demands. Feedback: Physical exertion increases the myocardial oxygen demand. If the patient has arteriosclerosis of the coronary arteries, then blood supply is diminished to the myocardium. Exercise does not cause vasoconstriction or interfere with drug metabolism. Exercise does not shunt blood flow away from the heart.

The nurse has just admitted a 66-year-old patient for cardiac surgery. The patient tearfully admits to the nurse that she is afraid of dying while undergoing the surgery. What is the nurse's best response? A) Explore the factors underlying the patient's anxiety. B) Teach the patient guided imagery techniques. C) Obtain an order for a PRN benzodiazepine. D) Describe the procedure in greater detail.

Ans: Explore the factors underlying the patient's anxiety. Feedback: An assessment of anxiety levels is required in the patient to assist the patient in identifying fears and developing coping mechanisms for those fears. The nurse must further assess and explore the patient's anxiety before providing interventions such as education or medications.

A nurse is caring for a patient post cardiac surgery. Upon assessment, the patient appears restless and is complaining of nausea and weakness. The patient's ECG reveals peaked T waves. The nurse reviews the patient's serum electrolytes anticipating which of the following abnormalities? a) Hyperkalemia b) Hyponatremia c) Hypercalcemia d) Hypomagnesemia

Ans: Hyperkalemia Feedback: Hyperkalemia is indicated by mental confusion, restlessness, nausea, weakness, and dysrhythmias (tall, peaked T waves). Hypercalcemia would likely be demonstrated by asystole. Hypomagnesemia would likely be demonstrated by hypotension, lethargy, and vasodilation. Hyponatremia may be indicated by weakness, fatigue, and confusion without change in T-wave formation.

A new surgical patient who has undergone a coronary artery bypass graft (CABG) is receiving opioids for pain control. The nurse must be alert to adverse effects of opioids. Which of the following effects would be important for the nurse to document? A) Urinary incontinence B) Hyperactive bowel sounds C) Hypotension D) Hypertension

Ans: Hypotension Feedback: The patient is observed for any adverse effects of opioids, which may include respiratory depression, hypotension, ileus, or urinary retention. If serious side effects occur, an opioid antagonist, such as Narcan, may be used.

Which of the following is the most important postoperative assessment parameter for patients undergoing cardiac surgery? a) Mental alertness b) Blood glucose level c) Inadequate tissue perfusion d) Activity intolerance

Ans: Inadequate tissue perfusion Feedback: The nurse must assess the patient for signs and symptoms of inadequate tissue perfusion, such as a weak or absent pulse, cold or cyanotic extremities, or mottling of the skin. Although the nurse does assess blood sugar and mental status, tissue perfusion is the higher priority. Assessing for activity intolerance, while important later in the recovery period, is not essential in the immediate postoperative period for patients undergoing cardiac surgery.

The nurse is caring for a patient who has undergone percutaneous transluminal coronary angioplasty (PTCA). What is the major indicator of success for this procedure? A) Increase in the size of the artery's lumen B) Decrease in arterial blood flow in relation to venous flow C) Increase in the patient's resting heart rate D) Increase in the patient's level of consciousness (LOC)

Ans: Increase in the size of the artery's lumen Feedback: PTCA is used to open blocked coronary vessels and resolve ischemia. The procedure may result in beneficial changes to the patient's LOC or heart rate, but these are not the overarching goals of PTCA. Increased arterial flow is the focus of the procedures.

The nurse is caring for an adult patient who had symptoms of unstable angina upon admission to the hospital. What nursing diagnosis underlies the discomfort associated with angina? A) Ineffective breathing pattern related to decreased cardiac output B) Anxiety related to fear of death C) Ineffective cardiopulmonary tissue perfusion related to coronary artery disease (CAD) D) Impaired skin integrity related to CAD

Ans: Ineffective cardiopulmonary tissue perfusion related to coronary artery disease (CAD) Feedback: Ineffective cardiopulmonary tissue perfusion directly results in the symptoms of discomfort associated with angina. Anxiety and ineffective breathing may result from angina chest pain, but they are not the causes. Skin integrity is not impaired by the effects of angina.

The nurse is assessing a patient who was admitted to the critical care unit 3 hours ago following cardiac surgery. The nurse's most recent assessment reveals that the patient's left pedal pulses are not palpable and that the right pedal pulses are rated at +2. What is the nurse's best response? A) Document this expected assessment finding during the initial postoperative period. B) Reposition the patient with his left leg in a dependent position. C) Inform the patient's physician of this assessment finding. D) Administer an ordered dose of subcutaneous heparin.

Ans: Inform the patient's physician of this assessment finding. Feedback: If a pulse is absent in any extremity, the cause may be prior catheterization of that extremity, chronic peripheral vascular disease, or a thromboembolic obstruction. The nurse immediately reports newly identified absence of any pulse.

Preoperative education is an important part of the nursing care of patients having coronary artery revascularization. When explaining the pre- and postoperative regimens, the nurse would be sure to include education about which subject? A) Symptoms of hypovolemia B) Symptoms of low blood pressure C) Complications requiring graft removal D) Intubation and mechanical ventilation

Ans: Intubation and mechanical ventilation Feedback: Most patients remain intubated and on mechanical ventilation for several hours after surgery. It is important that patients realize that this will prevent them from talking, and the nurse should reassure them that the staff will be able to assist them with other means of communication. Teaching would generally not include symptoms of low blood pressure or hypovolemia, as these are not applicable to most patients. Teaching would also generally not include rare complications that would require graft removal.

The triage nurse in the ED assesses a 66-year-old male patient who presents to the ED with complaints of midsternal chest pain that has lasted for the last 5 hours. If the patient's symptoms are due to an MI, what will have happened to the myocardium? A) It may have developed an increased area of infarction during the time without treatment. B) It will probably not have more damage than if he came in immediately. C) It may be responsive to restoration of the area of dead cells with proper treatment. D) It has been irreparably damaged, so immediate treatment is no longer necessary.

Ans: It may have developed an increased area of infarction during the time without treatment. Feedback: When the patient experiences lack of oxygen to myocardium cells during an MI, the sooner treatment is initiated, the more likely the treatment will prevent or minimize myocardial tissue necrosis. Delays in treatment equate with increased myocardial damage. Despite the length of time the symptoms have been present, treatment needs to be initiated immediately to minimize further damage. Dead cells cannot be restored by any means.

The nurse is caring for a patient who has been diagnosed with an elevated cholesterol level. The nurse is aware that plaque on the inner lumen of arteries is composed chiefly of what? A) Lipids and fibrous tissue B) White blood cells C) Lipoproteins D) High-density cholesterol

Ans: Lipids and fibrous tissue Feedback: As T-lymphocytes and monocytes infiltrate to ingest lipids on the arterial wall and then die, a fibrous tissue develops. This causes plaques to form on the inner lumen of arterial walls. These plaques do not consist of white cells, lipoproteins, or high-density cholesterol.

The nurse is providing care for a patient with high cholesterol and triglyceride values. In teaching the patient about therapeutic lifestyle changes such as diet and exercise, the nurse realizes that the desired goal for cholesterol levels is which of the following? A) High HDL values and high triglyceride values B) Absence of detectable total cholesterol levels C) Elevated blood lipids, fasting glucose less than 100 D) Low LDL values and high HDL values

Ans: Low LDL values and high HDL values Feedback: The desired goal for cholesterol readings is for a patient to have low LDL and high HDL values. LDL exerts a harmful effect on the coronary vasculature because the small LDL particles can be easily transported into the vessel lining. In contrast, HDL promotes the use of total cholesterol by transporting LDL to the liver, where it is excreted. Elevated triglycerides are also a major risk factor for cardiovascular disease. A goal is also to keep triglyceride levels less than 150 mg/dL. All individuals possess detectable levels of total cholesterol.

Which discharge instruction for self-care should the nurse provide to a client who has undergone a percutaneous transluminal coronary angioplasty (PTCA) procedure? A) Cleanse the site with disinfectants and dress the wound appropriately B) Refrain from sexual activity for 1 month C) Normal activities of daily living can be resumed the first day after surgery D) Monitor the site for bleeding or hematoma.

Ans: Monitor the site for bleeding or hematoma. Feedback: The nurse provides certain discharge instructions for self-care, such as monitoring the site for bleeding or the development of a hard mass indicative of hematoma. A nurse does not advise the client to clean the site with disinfectants or refrain from sexual activity for 1 month.

An adult patient is admitted to the ED with chest pain. The patient states that he had developed unrelieved chest pain that was present for approximately 20 minutes before coming to the hospital. To minimize cardiac damage, the nurse should expect to administer which of the following interventions? A) Thrombolytics, oxygen administration, and nonsteroidal anti-inflammatories B) Morphine sulphate, oxygen, and bed rest C) Oxygen and beta-adrenergic blockers D) Bed rest, albuterol nebulizer treatments, and oxygen

Ans: Morphine sulphate, oxygen, and bed rest Feedback: The patient with suspected MI should immediately receive supplemental oxygen, aspirin, nitroglycerin, and morphine. Morphine sulphate reduces preload and decreases workload of the heart, along with increased oxygen from oxygen therapy and bed rest. With decreased cardiac demand, this provides the best chance of decreasing cardiac damage. NSAIDs and beta-blockers are not normally indicated. Albuterol, which is a medication used to manage asthma and respiratory conditions, will increase the heart rate.

A client presents to the ED reporting anxiety and chest pain after shoveling heavy snow that morning. The client says that nitroglycerin has not been taken for months but upon experiencing this chest pain did take three nitroglycerin tablets. Although the pain has lessened, the client states, "They did not work all that well." The client shows the nurse the nitroglycerin bottle; the prescription was filled 12 months ago. The nurse anticipates which order by the physician? a) Serum electrolytes b) Nitroglycerin SL c) Chest x-ray d) Ativan 1 mg orally

Ans: Nitroglycerin SL Feedback: Nitroglycerin is volatile and is inactivated by heat, moisture, air, light, and time. Nitroglycerin should be renewed every 6 months to ensure full potency. The client's tablets were expired, and the nurse should anticipate administering nitroglycerin to assess whether the chest pain subsides. The other choices may be ordered at a later time, but the priority is to relieve the client's chest pain.

An ED nurse is assessing an adult woman for a suspected MI. When planning the assessment, the nurse should be cognizant of what signs and symptoms of MI that are particularly common in female patients? Select all that apply. A) Shortness of breath B) Chest pain C) Anxiety D) Numbness E) Weakness

Ans: Numbness, Weakness Feedback: Although these symptoms are not wholly absent in men, many women have been found to have atypical symptoms of MI, including indigestion, nausea, palpitations, and numbness. Shortness of breath, chest pain, and anxiety are common symptoms of MI among patients of all ages and genders.

The nurse is providing an educational workshop about coronary artery disease (CAD) and its risk factors. The nurse explains to participants that CAD has many risk factors, some that can be controlled and some that cannot. What risk factors would the nurse list that can be controlled or modified? A) Gender, obesity, family history, and smoking B) Inactivity, stress, gender, and smoking C) Obesity, inactivity, diet, and smoking D) Stress, family history, and obesity

Ans: Obesity, inactivity, diet, and smoking Feedback: The risk factors for CAD that can be controlled or modified include obesity, inactivity, diet, stress, and smoking. Gender and family history are risk factors that cannot be controlled.

A patient is recovering in the hospital from cardiac surgery. The nurse has identified the diagnosis of risk for ineffective airway clearance related to pulmonary secretions. What intervention best addresses this risk? A) Administration of bronchodilators by nebulizer B) Administration of inhaled corticosteroids by metered dose inhaler (MDI) C) Patient's consistent performance of deep breathing and coughing exercises D) Patient's active participation in the cardiac rehabilitation program

Ans: Patient's consistent performance of deep breathing and coughing exercises Feedback: Clearance of pulmonary secretions is accomplished by frequent repositioning of the patient, suctioning, and chest physical therapy, as well as educating and encouraging the patient to breathe deeply and cough. Medications are not normally used to achieve this goal. Rehabilitation is important, but will not necessarily aid the mobilization of respiratory secretions.

The nurse is caring for patient who tells the nurse that he has an angina attack beginning. What is the nurse's most appropriate initial action? A) Have the patient sit down and put his head between his knees. B) Have the patient perform pursed-lip breathing. C) Have the patient stand still and bend over at the waist. D) Place the patient on bed rest in a semi-Fowler's position.

Ans: Place the patient on bed rest in a semi-Fowler's position. Feedback: When a patient experiences angina, the patient is directed to stop all activities and sit or rest in bed in a semi-Fowler's position to reduce the oxygen requirements of the ischemic myocardium. Pursed-lip breathing and standing will not reduce workload to the same extent. No need to have the patient put his head between his legs because cerebral perfusion is not lacking.

A patient presents to the ED in distress and complaining of "crushing" chest pain. What is the nurse's priority for assessment? A) Prompt initiation of an ECG B) Auscultation of the patient's point of maximal impulse (PMI) C) Rapid assessment of the patient's peripheral pulses D) Palpation of the patient's cardiac apex

Ans: Prompt initiation of an ECG Feedback: The 12-lead ECG provides information that assists in ruling out or diagnosing an acute MI. It should be obtained within 10 minutes from the time a patient reports pain or arrives in the ED. Each of the other listed assessments is valid, but ECG monitoring is the most time dependent priority.

A client has had oral anticoagulation ordered. What should the nurse monitor for when the client is taking oral anticoagulation? a) Prothrombin time (PT) or international normalized ratio (INR) b) Hourly IV infusion c) Vascular sites for bleeding d) Urine output

Ans: Prothrombin time (PT) or international normalized ratio (INR) Feedback: The nurse should monitor PT or INR when oral anticoagulation is prescribed. Vascular sites for bleeding, urine output, and hourly IV infusions are generally monitored in all clients.

A patient with cardiovascular disease is being treated with amlodipine (Norvasc), a calcium channel blocking agent. The therapeutic effects of calcium channel blockers include which of the following? A) Reducing the heart's workload by decreasing heart rate and myocardial contraction B) Preventing platelet aggregation and subsequent thrombosis C) Reducing myocardial oxygen consumption by blocking adrenergic stimulation to the heart D) Increasing the efficiency of myocardial oxygen consumption, thus decreasing ischemia and relieving pain

Ans: Reducing the heart's workload by decreasing heart rate and myocardial contraction Feedback: Calcium channel blocking agents decrease sinoatrial node automaticity and atrioventricular node conduction, resulting in a slower heart rate and a decrease in the strength of the heart muscle contraction. These effects decrease the workload of the heart. Antiplatelet and anticoagulation medications are administered to prevent platelet aggregation and subsequent thrombosis, which impedes blood flow. Beta-blockers reduce myocardial consumption by blocking beta-adrenergic sympathetic stimulation to the heart. The result is reduced myocardial contractility (force of contraction) to balance the myocardium oxygen needs and supply. Nitrates reduce myocardial oxygen consumption, which decreases ischemia and relieves pain by dilating the veins and, in higher doses, the arteries.

The nurse is caring for a patient who is believed to have just experienced an MI. The nurse notes changes in the ECG of the patient. What change on an ECG most strongly suggests to the nurse that ischemia is occurring? A) P wave inversion B) T wave inversion C) Q wave changes with no change in ST or T wave D) P wave enlargement

Ans: T wave inversion Feedback: T-wave inversion is an indicator of ischemic damage to myocardium. Typically, few changes to P waves occur during or after an MI, whereas Q-wave changes with no change in the ST or T wave indicate an old MI.

A nurse is evaluating a client who had a myocardial infarction (MI) 7 days earlier. Which outcome indicates that the client is responding favorably to therapy? A) The client exhibits a heart rate above 100 beats/minute. B) The client verbalizes the intention of making all necessary lifestyle changes except for stopping smoking. C) The client states that sublingual nitroglycerin usually relieves his chest pain. D) The client demonstrates ability to tolerate more activity without chest pain.

Ans: The client demonstrates ability to tolerate more activity without chest pain. Feedback: The ability to tolerate more activity without chest pain indicates a favorable response to therapy in a client who is recovering from an MI or who has a history of coronary artery disease. The client should have a normal electrocardiogram with no arrhythmias and a regular heart rate of 60 to 100 beats/minute. Smoking is a cardiovascular risk factor that the client must be willing to eliminate. A client who responds favorably to therapy shouldn't have chest pain.

The nurse is working with a patient who had an MI and is now active in rehabilitation. The nurse should teach this patient to cease activity if which of the following occurs? A) The patient experiences chest pain, palpitations, or dyspnea. B) The patient experiences a noticeable increase in heart rate during activity. C) The patient's oxygen saturation level drops below 96%. D) The patient's respiratory rate exceeds 30 breaths/min.

Ans: The patient experiences chest pain, palpitations, or dyspnea. Feedback: Any activity or exercise that causes dyspnea and chest pain should be stopped in the patient with CAD. Heart rate must not exceed the target rate, but an increase above resting rate is expected and is therapeutic. In most patients, a respiratory rate that exceeds 30 breaths/min is not problematic. Similarly, oxygen saturation slightly below 96% does not necessitate cessation of activity.

When assessing a patient diagnosed with angina pectoris it is most important for the nurse to gather what information? A) The patient's activities limitations and level of consciousness after the attacks B) The patient's symptoms and the activities that precipitate attacks C) The patient's understanding of the pathology of angina D) The patient's coping strategies surrounding the attacks

Ans: The patient's symptoms and the activities that precipitate attacks Feedback: The nurse must gather information about the patient's symptoms and activities, especially those that precede and precipitate attacks of angina pectoris. The patient's coping, understanding of the disease, and status following attacks are all important to know, but causative factors are a primary focus of the assessment interview.

Family members bring a patient to the ED with pale cool skin, sudden midsternal chest pain unrelieved with rest, and a history of CAD. How should the nurse best interpret these initial data? A) The symptoms indicate angina and should be treated as such. B) The symptoms indicate a pulmonary etiology rather than a cardiac etiology. C) The symptoms indicate an acute coronary episode and should be treated as such. D) Treatment should be determined pending the results of an exercise stress test.

Ans: The symptoms indicate an acute coronary episode and should be treated as such. Feedback: Angina and MI have similar symptoms and are considered the same process, but are on different points along a continuum. That the patient's symptoms are unrelieved by rest suggests an acute coronary episode rather than angina. Pale cool skin and sudden onset are inconsistent with a pulmonary etiology. Treatment should be initiated immediately regardless of diagnosis.

The patient has had biomarkers drawn after complaining of chest pain. Which diagnostic of myocardial infarction remains elevated for as long as 2-3 weeks? a) Troponin b) CK-MB c) Myoglobin d) Total creatine kinase

Ans: Troponin Feedback: Troponin remains elevated for as long as 2-3 weeks, and it therefore can be used to detect recent myocardial damage. Myoglobin returns to normal in 12 hours. Total CK returns to normal in 3 days. CK-MB returns to normal in 3 to 4 days.

The nurse is caring for a patient who is scheduled for cardiac surgery. What should the nurse include in preoperative care? A) With the patient, clarify the surgical procedure that will be performed. B) Withhold the patient's scheduled medications for at least 12 hours preoperatively. C) Inform the patient that health teaching will begin as soon as possible after surgery. D) Avoid discussing the patient's fears as not to exacerbate them.

Ans: With the patient, clarify the surgical procedure that will be performed. Feedback: Preoperatively, it is necessary to evaluate the patient's understanding of the surgical procedure, informed consent, and adherence to treatment protocols. Teaching would begin on admission or even prior to admission. The physician would write orders to alter the patient's medication regimen if necessary; this will vary from patient to patient. Fears should be addressed directly and empathically.


Ensembles d'études connexes

History of Psychology Video Notes

View Set

Parts of the brain and what each does

View Set

Help desk Chapter 6 Help Desk Operation

View Set

Chapter 15 Concept recall questions

View Set

Windows Server 2012 Exam 2, 2. Active Directory Administration, Windows Server Chapter 6, 70-640 : Windows Server 2008 Active Directory Configuration (Ch.1), Windows Server Chapter 6, Chapter 6 Terminology, Tools 4.2

View Set

Vistas 2, Chapter 7, (12) 4 - Escoger, Online Homework, Reflexive Verbs

View Set

World Geography Spring Interim Review 30a-35e

View Set

The Expenditures Approach (LM 6 Part 1)

View Set